Soluções Física - Semana 79

Escrito por Matheus Ponciano

Iniciante:

Situação Física

Ocorreu uma colisão unidimensional entre dois blocos de forma que a velocidade relativa final entre elas fosse zero. Analisaremos diversos parâmetros dessa situação.

[collapse]
Solução

a) Tendo o coeficiente elástico dado por e=\frac{|v_{afastamento}|}{|v_{aproximacao}|} . Como ambos andam juntos após a colisão, temos que: v_{afastamento} = 0

e = 0

A colisão é então inelástica.

b) Como não atuam forças resultantes externas ao sistema dos bloquinhos, a quantidade de momento se conserva.

c) Conservando o momento:

m_aV_a + m_bV_b = (m_a+m_b)V

V=(m_aV_a + m_bV_b)/(m_a + m_b)

Vendo as energias cinéticas antes e depois da colisão:

E_{antes} = \frac{m_aV_a^2}{2} + \frac{m_bV_b^2}{2}

E_{depois`} = \frac{(m_a + m_b)V^2}{2}

E_{depois} = (m_a + m_b) \frac{(m_aV_a + m_bV_b)^2}{2(m_a + m_b)^2}

E_{depois} = \frac{(m_aV_a + m_bV_b)^2}{2(m_a + m_b)}

 

E_{antes} = \frac{(m_aV_a^2 + m_bV_b^2)}{2} \frac{(m_a + m_b)}{(m_a+m_b)}

E_{antes} = \frac{m_a^2V_a^2 +m_am_bV_a^2 + m_am_bV_b^2 +m_b^2V_b^2}{2(m_a + m_b)}

E_{antes} = \frac{m_a^2V_a^2 + 2m_am_bV_aV_b + m_b^2V_b^2 +m_am_bV_a^2 - 2m_am_bV_aV_b +m_am_bV_b^2}{2(m_a + m_b)}

E_{antes} = \frac{(m_aV_a +m_bV_b)^2 +m_am_b(V_a - V_b)^2}{2(m_a+m_b)}

 

Como a E_{antes} apresenta o termo \frac{m_am_b(V_a - V_b)^2}{2(m_a + m_b)} a mais que E_{depois}, a Energia cinética diminui após a colisão.

[collapse]
Gabarito

a) A colisão é inelástica

b) A quantidade de movimento se conserva

c) A energia não se conserva.

[collapse]

Intermediário:

Situação Física

Nessa questão analisaremos uma situação de equilíbrio térmico, ou seja, uma situação em que a temperatura de nenhum ponto do sistema variará mais com o tempo. Em seguida, tornaremos isso um problema relacionado com calor Latente de fusão.

[collapse]
Solução

a) Admitindo-se o fluxo estacionário de calor, temos que:

\Phi _{Al} = \Phi _{Cu}

\frac{K_{Al}A(T_A - T)}{L_{Al}} = \frac{K_{Cu}A(T-T_B)}{L_{Cu}}

\frac{0,48(100 - T)}{5} = \frac{0,92(T - 0)}{10}

0,96(100 - T) = 0,92T

96 - 0,96T = 0,92T

1,88T = 96

T = \frac{96}{1,88}

T = 51^{ o }  C

b) Pegando o fluxo de calor na barra:

\Phi = \frac{K_{Cu}A(T - T_B)}{L_{Cu}}

\Phi = \frac{0,92*1*(51 - 0)}{10}

\Phi = 4,7 \frac{cal}{s}

Temos que:

\Phi = \frac{Q}{\Delta t}

Q =\Phi \Delta t

Mas Q é a energia usada para fundir o gelo, logo:

Q = m_{fundida}L_{fusao}

m_{fundida}L_{fusao}= \Phi \Delta t

m_{fundida} = \frac{\Phi \Delta t}{L_{fusao}}

Substituindo os valores:

m_{fundida} = \frac{4,7*3600}{80}

m_{fundida} = 211,5 g

[collapse]
Gabarito

a) T=51 ^{\circle} C

b) m_{fundida}=211,5 g

[collapse]

Avançado:

Situação Física

A situação Física é bem simples. Temos uma partícula que não sofre uma força resultante externa e queremos provar a partir da segunda lei de Newton que essa se move em linha reta, entretanto, queremos fazer isso em coordenadas polares.

[collapse]
Solução

Como nenhuma força atua na partícula, suas acelerações radial e tangencial são 0, logo:

0 = \ddot r - \dot \theta ^2 r

0 = 2\dot r \dot \theta + \ddot \theta r

Com isso:

\ddot r = \dot \theta ^2 r

 

-2\dot r \dot \theta = \ddot \theta r

-2 \frac{d \dot r }{dt} \theta = \frac{ d \dot \theta}{d t} r

-2 \frac{d r}{r} = \frac{d \dot \theta}{\theta}

-2 \int_{r_o}^r \frac{dr}{r} = \int_{\dot \theta _o }^{\dot \theta} \frac{d \dot \theta}{\dot \theta}

ln(\frac{r_o}{r})^2 = ln(\frac{\dot \theta}{\dot \theta _o}

(\frac{r_o}{r})^2 = \frac{\dot \theta}{\dot \theta _o}

\dot \theta = \frac{ \dot \theta _o r_o^2}{r^2}

Substituindo na aceleração radial:

\ddot r = \frac{\dot \theta _o^2 r_o^4}{r^4} r

\dot r \frac{d \dot r}{d t} = \frac{\dot \theta _o^2 r_o^4}{r^3}

\dot r d \dot r = \dot \theta _o^2 r_o^4 \frac{d r}{r^3}

\int_{\dot r_o}^{\dot r} \dot r d \dot r = \dot \theta _o^2 r_o^4 \int_{r_o}^{r} \frac{d r}{r^3}

\frac{\dot r ^2 - \dot r _o^2}{2} = -\frac{\dot \theta _o^2 r_o^4}{2} (\frac{1}{r^2} - \frac{1}{r_o^2})

Como r_o é definido a partir do ponto onde ele fica mais próximo da origem, temos que \dot r_o = 0, por neste ponto ocorrer a inversão da velocidade, logo:

\dot r ^2 = \dot \theta _o^2 r_o^2 - \frac{\dot \theta _o^2 r_o^4}{r^2}

\dot r = (\dot \theta _o^2 r_o^2 - \frac{\dot \theta _o^2 r_o^4}{r^2})^{\frac{1}{2}}

\dot r = \dot \theta _o r_o (1 - \frac{r_o^2}{r^2})^{\frac{1}{2}}

\frac{dr}{dt} = \dot \theta _o r_o \sqrt{1 - \frac{r_o^2}{r^2}}

\frac{dr}{(1 - \frac{r_o^2}{r^2})^{1/2}} = \dot \theta _o r_o dt

\int_{r_o}^{r} \frac{dr}{\sqrt{1 - \frac{r_o^2}{r^2}}}=\int_0^t \dot \theta _o r_o dt

\int_{r_o}^{r} \frac{rdr}{\sqrt{r^2 - r_o^2}} = \dot \theta _o r_o t

Fazendo uma mudança de variável, chamando: u = r^2 - r_o^2    du = 2rdr

\int_{0}^{r^2 - r_o^2} \frac{du}{2\sqrt{u}} = \dot \theta _o r_o t

\frac{2(r^2 -r)o^2)^{\frac{1}{2}}}{2} = \dot \theta _o r_o t

r^2 - r_o^2 = \dot \theta _o^2 r_o^2 t^2

r = (r_o^2 + r_o^2 \dot \theta _o^2 t^2)^{\frac{1}{2}}

r = r_o(1 + \dot \theta _o^2 t^2)^{\frac{1}{2}}

Agora com r(t) podemos substituir na fórmula do \dot \theta para encontrar \theta (t).

\dot \theta = \frac{\dot \theta _o r_o^2}{r^2}

\dot \theta = \frac{\dot \theta _o r_o^2}{ r_o^2(1 + \dot \theta _o^2 t^2)}

\frac{d \theta}{dt} = \frac{\dot \theta _o}{ 1 + \dot \theta_o^2 t^2}

d \theta = \frac{\dot \theta _o dt}{1 + \dot \theta_o^2 t^2}

Fazendo uma mudança de variável, chamando: v = \dot \theta _o t     dv = \dot \theta _o dt

d \theta = \frac{dv}{1 + v^2}

Outra mudança de variável, com: \tan(\alpha) = v     \frac{d \alpha}{\cos^2(\alpha)} = dv

d \theta = \frac{d \alpha}{\cos^2(\alpha)} \frac{1}{1 + \tan^2(\alpha)}

Como 1 + \tan^2(\alpha) = \frac{1}{\cos^2(\alpha)}

d \theta = d \alpha

\int_0^{\theta} d \theta = \int_0^{\arctan (\dot \theta _o t)} d \alpha

\theta = \arctan(\dot \theta _o t)

\tan(\theta) = \dot \theta _o t

Mas pela equação de r(t) tiramos que \dot \theta _o t = (\frac{r^2}{r_o^2} -1)^{\frac{1}{2}}, logo:

(\frac{1}{\cos^2(\theta)})^{\frac{1}{2}} = (\frac{r^2}{r_o^2} -1)^{\frac{1}{2}}

\frac{1}{\cos^2(\theta)} - 1 =\frac{r^2}{r_o^2} -1

\cos^2(\theta) = \frac{r_o^2}{r^2}

\cos(\theta) = \frac{r_o}{r}

[collapse]
Gabarito

Demonstração.

[collapse]